Question

represent the simplest reduction of (ab' + ac)' a) a' + bc' b) a' + b'c...

represent the simplest reduction of
(ab' + ac)'

a) a' + bc'
b) a' + b'c
c) a + bc'
d) a + bc

Homework Answers

Answer #1

Given function is : (ab' + ac)'

=> (ab' + ac)'
=>(ab')' (ac)' (A+B)' = A'.B' - de Morgan’s Theorem

=>(a' + (b')') (a' + c') (A.B)' = A'+ B' - de Morgan’s Theorem
=>(a'+b) (a'+c') (A')' = A Double Negation Law
=> a'a' + ba'+a'c'+ bc'
=> a' + ba' + a'c' + bc' A.A = A   Idempotent Law
=> a'(1+b) + a'c' + bc' A + 1 = 1 Annulment Law
=> a'+ a'c' + bc'
=>a'(1+c') + bc' A + 1 = 1 Annulment Law
=> a'+ bc'

So anwer is : a) a'+ bc'   

Know the answer?
Your Answer:

Post as a guest

Your Name:

What's your source?

Earn Coins

Coins can be redeemed for fabulous gifts.

Not the answer you're looking for?
Ask your own homework help question
Similar Questions
Prove that if A*B*C, then ray AB = ray AC and ray BC is a subset...
Prove that if A*B*C, then ray AB = ray AC and ray BC is a subset of ray AC
Suppose △ABC and △A'B'C' are triangles such that line AB || to line A'B', line BC...
Suppose △ABC and △A'B'C' are triangles such that line AB || to line A'B', line BC || to line B'C', and line AC || to line A'C'. Prove that △ABC ~ △A'B'C'.
i have the following problem boolean simplfication A'BC' + BC + AB'C' +AB'C I can do...
i have the following problem boolean simplfication A'BC' + BC + AB'C' +AB'C I can do it two different ways factor AB' out of the 3rd and 4rth terms giving me A'B'C' +BC + AB'(C'+C) then using the boolean rules that says C'+ C =1 and the rule that says anything anded with 1 is it's self i have A'B'C' + BC + AB' comuntive law AB' + A'B'C + BC now factoring B' out of the first two terms...
In △ABC, AB = 15 cm, BC = 10 cm, and AC = 6 cm. Find...
In △ABC, AB = 15 cm, BC = 10 cm, and AC = 6 cm. Find the measure of angle B to the nearest degree.
Suppose that the incircle of triangle ABC touches AB at Z, BC at X, and AC...
Suppose that the incircle of triangle ABC touches AB at Z, BC at X, and AC at Y . Show that AX, BY , and CZ are concurrent.
Prove that given △ABC and △A′B′C′, if we have AB ≡ A′B′ and BC≡B′C′,then B<B′ if...
Prove that given △ABC and △A′B′C′, if we have AB ≡ A′B′ and BC≡B′C′,then B<B′ if and only if AC<A′C′. You cannot use measures.
Prove: P(A ∪ B ∪ C) = P(A) + P(Ac ∩ B) + P(Ac ∩ Bc...
Prove: P(A ∪ B ∪ C) = P(A) + P(Ac ∩ B) + P(Ac ∩ Bc ∩ C)
In the rectangle ABCD, AB = 6 and BC = 8. The diagonals AC and BD...
In the rectangle ABCD, AB = 6 and BC = 8. The diagonals AC and BD intersect at O. Point P lies on the diagonal AC such that AP = 1. A line is drawn from B through P and meets AD at S. Let be R a point on AD such that OR is parallel to BS. a) Find the lengths of AS and RD. Hint: Denote AS = x. Use P S k OR and OR k BS...
ABC is a right-angled triangle with right angle at A, and AB > AC. Let D...
ABC is a right-angled triangle with right angle at A, and AB > AC. Let D be the midpoint of the side BC, and let L be the bisector of the right angle at A. Draw a perpendicular line to BC at D, which meets the line L at point E. Prove that (a) AD=DE; and (b) ∠DAE=1/2(∠C−∠B) Hint: Draw a line from A perpendicular to BC, which meets BC in the point F
Give a mathematical derivation of the formula P((A ∩ Bc ) ∪ (Ac ∩ B)) =...
Give a mathematical derivation of the formula P((A ∩ Bc ) ∪ (Ac ∩ B)) = P(A) + P(B) − 2P(A ∩ B). Your derivation should be a sequence of steps, with each step justified by appealing to one of the probability axioms. ##### solution ########## 1 Since the events A ∩ Bc and Ac ∩ B are disjoint, we have, using the additivity axiom, P((A ∩ Bc ) ∪ (Ac ∩ B)) = P(A ∩ Bc ) + P(Ac...
ADVERTISEMENT
Need Online Homework Help?

Get Answers For Free
Most questions answered within 1 hours.

Ask a Question
ADVERTISEMENT